LSAT and Law School Admissions Forum

Get expert LSAT preparation and law school admissions advice from PowerScore Test Preparation.

User avatar
 Dave Killoran
PowerScore Staff
  • PowerScore Staff
  • Posts: 5849
  • Joined: Mar 25, 2011
|
#41159
Complete Question Explanation
(The complete setup for this game can be found here: lsat/viewtopic.php?t=11758)

The correct answer choice is (C)

As in previous questions, J and K play a key role in this question. If we need to place two students in such a way that the composition of every class is determined, we will need to address which of J and K will be in the level 1 class. Thus, the correct answer is likely to include one of J and K. Answer choices (B) and (E), neither of which include J or K, are therefore less likely to be correct.

Of the three remaining answers, we can apply a different analysis, but one that is equally effective. J and K are somewhat similar, and the three students in answer choice (A), (C), and (D) besides J and K are F, H, and L. In examining F, H, and L, note that F and L are virtually identical in their basic characteristics: both are at the end of the same chain, behind K. Thus, pairing K with F or L (as do (A) and (D)) is likely to produce a similar result. The Uniqueness Theory of Answer Choices suggests that two answers with similar characteristics are unlikely to be correct. On the other hand, H sits in the middle of a different chain and automatically impacts N, who has a lower score. Thus, H appears to be more unique than F and L, and for our purposes this suggests that answer choice (C) is most likely to be correct.

Let’s take a closer look at answer choice (C), then. If H and J are in the level 2 class, then M must be in the level 2 class with them. Consequently, K must be in the level 1 class, and N, F, and L must be in the level 3 class. Thus, answer choice (C) is correct.
 josuecarolina
  • Posts: 24
  • Joined: Jul 20, 2012
|
#9506
This is driving me mad. I selected D, the answer is C. They are both technically correct, I think, with either one you know which participants are in each group, but with D you actually know the order as well.

3 groups, 3 people in each group. POsition 1 and 2 of he first group are filled, position 3 must be J or K. Got that. So if K and L are in the second group, I know exactly where everyone else is...same as J and H...?
User avatar
 Dave Killoran
PowerScore Staff
  • PowerScore Staff
  • Posts: 5849
  • Joined: Mar 25, 2011
|
#9508
Hi Josue,

Good to hear from you again. this is a tough questions, and it sounds like you understand why (C) is correct. so, then, let's look at (D).

When K and L are placed in the level 2 class, we immediately know J is in the level 1 class:

..... ..... G I J ..... ..... K L
..... ..... Level 1 ..... Level 2 ..... Level 3

So, who is left? F, M, H, and N. Plus, we know M > H > N. But F has more freedom, and this is where answer (D) falls apart. While F must score lower than K, we do not know how F scores in relation to M, H, and N. Thus, F could be in the level 2 class, or F could be in the level 3 class. Because we are trying to find an answer that determines the composition of each class, and (D) does not allow us to completely do that, (D) must therefore be incorrect.

I'm thinking you were throwing F in the level 2 class, and then placing M > H > N in the level 3 class, but that assumes F > M, which is not known to be true.

Please let me know if that helps. Thanks!
 Cbarcelo
  • Posts: 9
  • Joined: Jul 30, 2018
|
#48960
Hey guys,

I got this question right but my analysis involved 0 understanding/knowledge of the "Uniqueness Theory of Answer Choices." Needless to say, I'm curious. I tried researching the theory but came back empty-handed. Aside from the brief explanation in the book, is there elsewhere I can read on it?

Best,
Claudia
User avatar
 Jonathan Evans
PowerScore Staff
  • PowerScore Staff
  • Posts: 726
  • Joined: Jun 09, 2016
|
#49022
Hi, Claudia,

Great question! The best existing explanation for the Uniqueness Rule of Answer Choices™ is in chapter 3 of the Logical Reasoning Bible (page 107). There are two important ideas intrinsic to this rule:
  1. The correct answer will satisfy the logical quality of the question stem, and the four incorrect answers will not. In addition to the three examples provided in the Bible, you could extrapolate this principle to any situation you encounter on the LSAT:
    • Correct answer: Resolve the paradox, Incorrect answers: Do not resolve the paradox, either no effect or make it worse
    • Correct answer: Statement with which the author would likely agree, Incorrect answers: No evidence author would agree, or evidence author would disagree
    • Correct answer: Cannot be true, Incorrect answers: Could be true, or possibly must be true
  2. If two answers are identical to each other, these two answers must be incorrect. These two incorrect answers could be literally identical, or they could have an identical logical quality. Either way they are incorrect.
You are very likely already implementing the principles illustrated in this rule! The Uniqueness Rule of Answer Choices is simply a conceptual framework with which we can better understand the mechanics of answers, both correct and incorrect, on the LSAT. Please follow up with further questions. I hope this helps!
 concrottrox11@gmail.com
  • Posts: 29
  • Joined: Dec 07, 2021
|
#92488
Hi,

I just read through the explanation for Q#4, but I still do not understand other than for the uniqueness theory of answer choices, why answers A, B, D, E are wrong?

Please let me know your thoughts. Thank you.
 Robert Carroll
PowerScore Staff
  • PowerScore Staff
  • Posts: 1783
  • Joined: Dec 06, 2013
|
#92506
concrottrox,

On the actual test, I'd say pick the answer that's provably correct and move on! Since you can prove (as above) that answer choice (C) is correct, and there can be only one correct answer per question, that's a proof that every other answer must have a problem. Practically, you have to get used to doing that - otherwise you're doing more work than you can fit into the 35 minutes of a Logic Game section.

So, while I'm about to prove that every other answer is wrong, if I were to do this on a real test, I'd seriously be ruining my chances of finishing the section - not good!

Going back to the initial post, answer choices (B) and (E) do not determine the placement of J or K. As the first post points out, that's problematic - Class 1 is always I, G, and one of (J/K). So answer choices (B) and (E) will probably each allow TWO solutions, one with J in Class 1 and one with K in Class 1:

Answer choice (B) allows both:

IGJ KFM LHN
IGK JFM LHN

Answer choice (E) allows both:

IGJ KLM FHN
IGK JLM FHN

Dave showed why answer choice (D) is wrong in the third post of this thread, so that's taken care of.

Answer choice (A) allows both:

IGJ KFM LHN
IGJ KFL MHN

Thus, we've proven that every answer but answer choice (C) allows at least two solutions, meaning they do not completely determine the composition.

Robert Carroll

Get the most out of your LSAT Prep Plus subscription.

Analyze and track your performance with our Testing and Analytics Package.